Last visit was: 26 Apr 2024, 21:31 It is currently 26 Apr 2024, 21:31

Close
GMAT Club Daily Prep
Thank you for using the timer - this advanced tool can estimate your performance and suggest more practice questions. We have subscribed you to Daily Prep Questions via email.

Customized
for You

we will pick new questions that match your level based on your Timer History

Track
Your Progress

every week, we’ll send you an estimated GMAT score based on your performance

Practice
Pays

we will pick new questions that match your level based on your Timer History
Not interested in getting valuable practice questions and articles delivered to your email? No problem, unsubscribe here.
Close
Request Expert Reply
Confirm Cancel
SORT BY:
Date
Tags:
Show Tags
Hide Tags
Math Expert
Joined: 02 Sep 2009
Posts: 92948
Own Kudos [?]: 619267 [3]
Given Kudos: 81609
Send PM
Most Helpful Reply
Math Expert
Joined: 02 Sep 2009
Posts: 92948
Own Kudos [?]: 619267 [0]
Given Kudos: 81609
Send PM
General Discussion
Director
Director
Joined: 26 Nov 2019
Posts: 799
Own Kudos [?]: 784 [1]
Given Kudos: 58
Location: South Africa
Send PM
Verbal Chat Moderator
Joined: 20 Mar 2018
Posts: 2001
Own Kudos [?]: 1614 [1]
Given Kudos: 1680
Send PM
Re: Mayor: Local antitobacco activists are calling for expanded antismokin [#permalink]
1
Kudos
Mayor: Local antitobacco activists are calling for expanded antismoking education programs paid for by revenue from heavily increased taxes on cigarettes sold in the city. Although the effectiveness of such education programs is debatable, there is strong evidence that the taxes themselves would produce the sought-after reduction in smoking. Surveys show that cigarette sales drop substantially in cities that impose stiff tax increases on cigarettes.

Which one of the following, if true, most undermines the reasoning in the argument above?


(A) A city-imposed tax on cigarettes will substantially reduce the amount of smoking in the city if the tax is burdensome to the average cigarette consumer. Incorrect

support argument reasoning

(B) Consumers are more likely to continue buying a product if its price increases due to higher taxes than if its price increases for some other reason. Incorrect

it contradicts with argument, cant explain why sales decreased but smoking cigarettes were not

(C) Usually, cigarette sales will increase substantially in the areas surrounding a city after that city imposes stiff taxes on cigarettes. Correct

so, sales decreased because of increased tax but consumption of cigarettes was not

(D) People who are well informed about the effects of long-term tobacco use are significantly less likely to smoke than are people who are not informed. Incorrect

talks about another fact

(E) Antismoking education programs that are funded by taxes on cigarettes will tend to lose their funding if they are successful. Incorrect

irrelevant
Senior Manager
Senior Manager
Joined: 23 Oct 2015
Posts: 323
Own Kudos [?]: 270 [1]
Given Kudos: 33
Location: United States (NH)
Concentration: Leadership, Technology
Schools: Wharton '25
WE:Information Technology (Non-Profit and Government)
Send PM
Re: Mayor: Local antitobacco activists are calling for expanded antismokin [#permalink]
1
Kudos
Since the taxes are local, the most undermining result would be
1. Educational program, though debatable, if proven that it is completely ineffective in control smoking.
2. Survey( that shows the sales is low with higher taxes) is some how incorrect. That is, people choose alternate options, by ordering online, drive out-of-city to buy the cigarette, choosing duty-free places like airport etc. Also, sales lower in the city does not mean that people of the city don't have other options.


(A) A city-imposed tax on cigarettes will substantially reduce the amount of smoking in the city if the tax is burdensome to the average cigarette consumer.
This is already known and does not weaken the argument. Eliminate

(B) Consumers are more likely to continue buying a product if its price increases due to higher taxes than if its price increases for some other reason.
This may or may not be true. If someone don't have enough money, they cannot buy the product, period. So this case is extreme and hence it does not qualify to undermine the argument. Eliminate.

(C) Usually, cigarette sales will increase substantially in the areas surrounding a city after that city imposes stiff taxes on cigarettes.
C, is the correct choice as per our analysis.

(D) People who are well informed about the effects of long-term tobacco use are significantly less likely to smoke than are people who are not informed.
well-informed people & effects of long-term tobacco use is irrelevant to the argument. Eliminate.

(E) Antismoking education programs that are funded by taxes on cigarettes will tend to lose their funding if they are successful.
It about eliminating smoking which is the true goal. If it is achieved then there is no need for antismoking programs at all. Eliminate.
Director
Director
Joined: 29 Apr 2019
Status:Learning
Posts: 751
Own Kudos [?]: 583 [1]
Given Kudos: 49
Send PM
Re: Mayor: Local antitobacco activists are calling for expanded antismokin [#permalink]
1
Kudos
Correct Option C

Mayor:
Local antitobacco activists are calling for expanded antismoking education programs
paid for by revenue from heavily increased taxes on cigarettes sold in the city.
Although the effectiveness of such education programs is debatable,
Evidence: taxes themselves would produce the sought-after reduction in smoking.
Surveys: cigarette sales drop substantially in cities that impose stiff tax increases on cigarettes.

Conclusion: Impose tax will drop tobacco sale
Assumption: considered only one option, that tobacco can be made available
Argument: People on tax impose on tobacco, will not purchase

Which one of the following, if true, most undermines the reasoning in the argument above?

(A) A city-imposed tax on cigarettes will substantially reduce the amount of smoking in the city
if the tax is burdensome to the average cigarette consumer.
Wrong: Strengthen the argument

(B) Consumers are more likely to continue buying a product
if its price increases due to higher taxes than
if its price increases for some other reason.
Weaken: to close to correct but, here condition can be implemented, what if price is high than expected

(C) Usually, cigarette sales will increase substantially in
the areas surrounding a city after that
city imposes stiff taxes on cigarettes.
Weaken: City impose tax sales decrease to nearby city where tax is not imposed sales increase, person might end up purchasing bulk amount at less price and consume as per need.

(D) People who are well informed
about the effects of long-term tobacco use are significantly less likely to smoke than are
people who are not informed.
Wrong: Informed vs non-Informed comparison between the people

(E) Antismoking education programs that are
funded by taxes on cigarettes will tend to lose them
funding if they are successful.
Wrong: irrelevant - gives detail about agency/ programs profit and loss
User avatar
Non-Human User
Joined: 01 Oct 2013
Posts: 17226
Own Kudos [?]: 848 [0]
Given Kudos: 0
Send PM
Re: Mayor: Local antitobacco activists are calling for expanded antismokin [#permalink]
Hello from the GMAT Club VerbalBot!

Thanks to another GMAT Club member, I have just discovered this valuable topic, yet it had no discussion for over a year. I am now bumping it up - doing my job. I think you may find it valuable (esp those replies with Kudos).

Want to see all other topics I dig out? Follow me (click follow button on profile). You will receive a summary of all topics I bump in your profile area as well as via email.
GMAT Club Bot
Re: Mayor: Local antitobacco activists are calling for expanded antismokin [#permalink]
Moderators:
GMAT Club Verbal Expert
6923 posts
GMAT Club Verbal Expert
238 posts
CR Forum Moderator
832 posts

Powered by phpBB © phpBB Group | Emoji artwork provided by EmojiOne